Taxicab Rooted Polynomial

Algebra Level 3

x ( x + 6 ) ( x + 12 ) ( x + 18 ) = 1729 \large x(x+6)(x+12)(x+18) = 1729

Find sum of all real roots of the above equation.


The answer is -18.

This section requires Javascript.
You are seeing this because something didn't load right. We suggest you, (a) try refreshing the page, (b) enabling javascript if it is disabled on your browser and, finally, (c) loading the non-javascript version of this page . We're sorry about the hassle.

11 solutions

Nihar Mahajan
Mar 3, 2015

x ( x + 6 ) ( x + 12 ) ( x + 18 ) = 1729 x(x+6)(x+12)(x+18) = 1729

x ( x + 18 ) ( x + 6 ) ( x + 12 ) = 1729 \Rightarrow x(x+18)(x+6)(x+12) = 1729

( x 2 + 18 x ) ( x 2 + 18 x + 72 ) = 1729 \Rightarrow (x^2+18x)(x^2 + 18x + 72) = 1729

Let x 2 + 18 x = a x^2 + 18x = a

a ( a + 72 ) = 1729 \Rightarrow a(a + 72) = 1729

a 2 + 72 a 1729 = 0 \Rightarrow a^2 + 72a - 1729 = 0

( a 19 ) ( a + 91 ) = 0 a = 19 o r 91 \Rightarrow (a-19)(a+91) = 0 \Rightarrow a = 19 \quad or \quad -91

When a = 91 x 2 + 18 x + 91 = 0 x = 18 ± 40 2 a = -91 \Rightarrow x^2 + 18x + 91 = 0 \Rightarrow x = \dfrac{-18 \pm \sqrt{-40}}{2} which is not possible since the question asks for real roots.

When a = 19 x 2 + 18 x = 19 x 2 + 18 x 19 = 0 ( x + 19 ) ( x 1 ) = 0 a = 19 \Rightarrow x^2 + 18x = 19 \Rightarrow x^2 + 18x - 19 = 0 \Rightarrow (x + 19)(x - 1) = 0

x = 1 , 19 \Rightarrow x = 1 , -19

Thus , sum real roots = 1 19 = 18 \huge=1-19 = \color{#D61F06}{ \boxed{-18}}

Alternative trial and error:

We can easily see that 1729 = 1 × 7 × 13 × 19 1729 = 1 \times 7 \times 13 \times 19 , where 1 , 7 , 13 , 19 1 , 7 , 13 , 19 form an Arithmetic progression with difference 6 6 .

Thus x = 1 , 19 x = 1 , -19

However , it is better to check whether there are other real roots existing or not

no need to do so much, 1st, x=u -9, ( u 9 ) ( u + 9 ) ( u 3 ) ( u + 3 ) = 1729 (u-9)(u+9)(u-3)(u+3)=1729 we form a bi-quadratic, where the sum of the real roots is always zero. we can deduce that the polynomial has 2 real by graphing. hence 2 ( x + 9 ) = 0 > 2 ( x ) = 2 9 = 18 \sum_{2} (x+9)=0-->\sum_{2} (x)=-2*9=\boxed{-18}

Aareyan Manzoor - 6 years, 3 months ago

Log in to reply

Woah. What just happened here. xD

Giwon Kim - 5 years ago

x ( x + 6 ) ( x + 12 ) ( x + 18 ) x(x+6)(x+12)(x+18) = 1729 1729

( x + 18 x ) ( x + 18 x + 72 ) (x+18x)(x+18x+72) = 1729 1729

let x 2 + 18 x + 36 x^2+18x+36 = a a

( a 36 ) ( a + 36 ) (a-36)(a+36) = a 2 1296 {a}^2-1296 = 1729 1729

a 2 a^2 = 3025 = 5 5 2 3025=55^2

x 2 + 18 x + 36 = ± 55 x^2+18x+36=\pm55

x 2 + 18 x + 36 = 55 x^2+18x+36=-55

x 2 + 18 x + 91 = 0 x^2+18x+91=0 There is no real solution

x 2 + 18 x + 36 = 55 x 2 + 18 x 19 = 0 x^2+18x+36=55 x^2+18x-19=0

There are two solutions. Sum of these two solutions is 18 -18

Altan-Ulzii Chuluun - 5 years, 10 months ago

You assumed x^2+18*x=a

Then you did a mistake in x^2+18*x+72=a+12.

It should be a+72.

Altan-Ulzii Chuluun - 5 years, 10 months ago

Log in to reply

please clarify whether its a error or not

Anupam Shandilya - 5 years, 9 months ago

Log in to reply

answer was correct. But there is a typo

Altan-Ulzii Chuluun - 5 years, 9 months ago

Nice skills

U Z - 6 years, 3 months ago

Log in to reply

To be fair, this question is pretty common. I've had a 7th grader give this question to me in the middle of my English exam. xD

Siddhartha Srivastava - 6 years, 3 months ago

Log in to reply

This exactly same one? I thought I was the first to frame this. :D :P

Nihar Mahajan - 6 years, 3 months ago

Log in to reply

@Nihar Mahajan I had done it using the Rational root test.The solutions to the polynomial are either ± 1 , ± 7 , ± 13 , ± 19 , ± 91 , ± 133 , ± 247 a n d ± 1729 \pm1,\pm7,\pm13,\pm 19,\pm 91,\pm 133,\pm 247 and \pm1729 .But by simple logic we need to check only the few couple of values,and hence we get 1 1 and 19 -19 .

Anik Mandal - 6 years, 3 months ago

Log in to reply

@Anik Mandal But by this test we get only rational roots. It is necessary to check whether irrational roots exist or not. Luckily in this problem there are no irrational roots.

Nihar Mahajan - 6 years, 3 months ago

Great Solution!

Aryan Gaikwad - 6 years, 3 months ago

Seriously forgot the minus sign. Stupid of me. Well a great solution

Aayush Patni - 6 years, 1 month ago

Well explained!

Thomas Siu - 6 years, 3 months ago

Outstanding, mind blowing and superhit problem!

Swapnil Das - 5 years, 10 months ago

Can I ask one thing, Why you subtracting 1 - 19 to get the sum, I am little confused

Syed Baqir - 5 years, 9 months ago

Log in to reply

1 , 19 1,-19 are the required roots. So their sum is 1 + ( 19 ) = 1 19 = 18 1+(-19)=1-19=-18 .

Nihar Mahajan - 5 years, 9 months ago

Log in to reply

Thank you, I was a little stupid there hehe :)

Syed Baqir - 5 years, 9 months ago

please explain the step: a^2+12a-1729=0 => (a-19)(a-91)=0

Ritwic Majumder - 5 years, 5 months ago
Caleb Townsend
Mar 3, 2015

Using Vieta's Formula
Find the first two coefficients of the expanded polynomial. They are 1 1 and 6 + 12 + 18 = 36. 6 + 12 + 18 = 36. By Vieta's formula, the sum of all roots is 36 1 = 36 \frac{-36}{1} = -36 but by looking at the graph, two roots aren't real.

By the complex conjugate root theorem, if a + b i a + bi is a root, then so is a b i . a - bi. Since the polynomial is symmetric about x = 9 , x = -9, the real roots are equidistant from 9 , -9, and the imaginary roots are equidistant from 9 , -9, and also equidistant from the real roots. Thus they are of the form 9 + b i -9 + bi and 9 b i . -9 - bi. Subtract their sum from 36 : -36: 36 ( 18 ) = 18 -36 - (-18) = \boxed{-18}

Using number theory
Looking at the graph, there are two real roots. Luckily, 1729 1729 's prime factors differ by the differences in the polynomial's factors: including 1 , 1, they are { 1 , 7 , 13 , 19 } . \{1,7,13,19\}. And since the polynomial is of even degree, we just need to find the case where the factors are all negative and the case where they're all positive. These are respectively x = 19 x = -19 and x = 1. x = 1. 1 19 = 18 1 - 19 = \boxed{-18}

Nice observation skills Caleb , I'll remember this method next time :)

A Former Brilliant Member - 6 years, 3 months ago

Its not actually 'luckily' . I used that fact to frame the question. :)

Nihar Mahajan - 6 years, 3 months ago
Chew-Seong Cheong
Jul 31, 2015

1729 = ( 1 ) ( 7 ) ( 13 ) ( 19 ) = ( 1 ) ( 7 ) ( 13 ) ( 19 ) = 1 ( 1 + 6 ) ( 1 + 12 ) ( 1 + 18 ) = ( 19 + 18 ) ( 19 + 12 ) ( 19 + 6 ) ( 19 ) \begin{aligned} 1729 & = \color{#3D99F6}{(1)(7) (13) (19)} = \color{#D61F06}{(-1)(-7) (-13) (-19)} \\ & = \color{#3D99F6}{1}(\color{#3D99F6}{1}+6)(\color{#3D99F6}{1}+12)(\color{#3D99F6}{1}+18) = (\color{#D61F06}{-19}+18) (\color{#D61F06}{-19}+12) (\color{#D61F06}{-19}+6) (\color{#D61F06}{-19}) \end{aligned}

The sum of real roots = 1 19 = 18 = 1 - 19 = \boxed{-18}

Moderator note:

This solution is incomplete. How do you that there can only be exactly 2 real roots? Why can't there be 3 or 4?

After factoring out the roots, you should check the discriminant of the remaining quadratic to see that it is negative.

Alan Yan - 5 years, 9 months ago
Willy Sumarno
Sep 30, 2015

Let y = x + 9 y=x+9 , then we have x ( x + 6 ) ( x + 12 ) ( x + 18 ) = ( y 9 ) ( y 3 ) ( y + 3 ) ( y + 9 ) = ( y 2 81 ) ( y 2 9 ) = y 4 90 y 2 + 729 = 1729 x(x+6)(x+12)(x+18)=(y-9)(y-3)(y+3)(y+9)=(y^2-81)(y^2-9)=y^4-90y^2+729=1729 y 4 90 y 2 1000 = 0 \Rightarrow y^4-90y^2-1000=0 ( y 2 100 ) ( y 2 + 10 ) = 0 \Rightarrow (y^2-100)(y^2+10)=0 ( y 10 ) ( y + 10 ) ( y 2 + 10 ) = 0 \Rightarrow (y-10)(y+10)(y^2+10)=0 Since y 2 + 10 = 0 y^2+10=0 don't have real roots, then we have y 1 = 10 y_1=10 and y 2 = 10 y_2=-10 . So, we have x 1 = 10 9 = 1 x_1=10-9=1 and x 2 = 10 9 = 19 x_2=-10-9=-19 , and the sum of all real roots of the equation above is x 1 + x 2 = 18 x_1+x_2=-18 .

Oli Hohman
Sep 22, 2015

Notice the prime factorization of 1729 = (7)(13)(19)= (91)(19) This makes it obvious x=1 is a solution

Using synthetic division with x=1, you find that the new cubic polynomial is x^3+37x^2+433x+1729 = 0

You can factor this further to be

(x+19) (x (x+18)+91) = 0

The number of imaginary solutions to any polynomial must be an even number. Since, (x(x+18)+91) = 0 only has imaginary solutions because the discriminant (b^2-4ac) < 0, you can conclude there are 2 real solutions and 2 complex solutions to the original quartic equation.

So, x=1,-19 are the only real solutions, so you have 1+ -19 = -18 as the answer.

Aravind Vishnu
Aug 5, 2015

1729 factors into 1729 = 7 × 13 × 19 hence the only possible solutions according to the above expression are 1729 = 1 × 7 × 13 × 19 OR 1729 = 1 × 7 × 13 × 19 and x = 1 or 19 1 19 = 18. 1729 \text{factors into} 1729=7\times 13\times 19 \\ \text{hence the only possible solutions according to the above expression are}\\ 1729=1\times 7\times 13\times 19 \\ \text{OR}\\ 1729=-1\times -7\times -13\times -19\\ \text{and } x=1 \text{ or } -19 \\ 1-19=-18.

Hitoshi Yamamoto
Jul 25, 2015

1729 = 7x13x19

Se x = 1

Logo, 1x7x13x19 = 1729

Induz o x+18 = -1, logo x = -19

x(x+6)(x+12)(x+18) = -19x(-13)x(-7)x(-1) = 1729

Foi pensado nessa possibilidade, pois são 4 raízes. Caso a função tenha números de raízes pares, logo o conjunto de valores negativos resultará num valor positivo.

Então se "x" pode ser -19 e 1. Somando, -19 + 1 = -18

Majed Kalaoun
Jun 20, 2017

x ( x + 6 ) ( x + 12 ) ( x + 18 ) = 1729 x(x+6)(x+12)(x+18)=1729

x ( x + 18 ) ( x + 6 ) ( x + 12 ) = 1729 x(x+18)(x+6)(x+12)=1729

( x 2 + 18 x ) ( x 2 + 18 x + 72 ) = 1729 (x^2+18x)(x^2+18x+72)=1729

Let n = ( x 2 + 18 x ) n=(x^2+18x)

n ( n + 72 ) = 1729 n(n+72)=1729

n 2 + 72 n 1729 = 0 n^2+72n-1729=0

( n 19 ) ( n + 91 ) = 0 (n-19)(n+91)=0 \Rightarrow n = 19 n=19 or n = 91 n=-91

When n = 91 n=-91 \Rightarrow x 2 + 18 x + 91 = 0 x^2+18x+91=0\Rightarrow x = 18 ± 40 2 x=\dfrac{-18\pm\sqrt{-40}}{2} . But the question asks for real roots, not complex ones, so this solution is irrelevant.

However, when n = 19 n=19\Rightarrow x 2 + 18 x = 19 x 2 + 18 x 19 = 0 ( x + 19 ) ( x 1 ) = 0 x^2+18x=19\Rightarrow x^2+18x-19=0\Rightarrow (x+19)(x-1)=0

x = 19 , x = 1 \Rightarrow x=-19, x=1

19 + 1 = 18 -19+1=18

József Inczefi
Aug 22, 2016

Let's say y = x + 9 , z = y 2 45 x ( x + 6 ) ( x + 12 ) ( x + 18 ) = 1729 ( y 9 ) ( y 3 ) ( y + 3 ) ( y + 9 ) = 1729 ( y 2 81 ) ( y 2 9 ) = 1729 ( z 36 ) ( z + 36 ) = 1729 z 2 1296 = 1729 z 2 = 3025 y 2 = ± 55 + 45 , y 2 = 10 o r y 2 = 100 y = ± 10 x = 1 o r x = 19 1 19 = 18 y=x+9,\quad z=y^{ 2 }-45\\ x(x+6)(x+12)(x+18)=1729\\ (y-9)(y-3)(y+3)(y+9)=1729\\ (y^{ 2 }-81)(y^{ 2 }-9)=1729\\ (z-36)(z+36)=1729\\ z^{ 2 }-1296=1729\\ z^{ 2 }=3025\\ y^{ 2 }=\pm 55+45,\quad y^{ 2 }=-10\quad or \quad y^{ 2 }=100\\ y=\pm 10\\ x=1\quad or\quad x=-19\\ 1-19=\boxed{ -18 }\\

I'm horrible at editing -_-

Ed Sirett
Aug 16, 2016

A bit of graph sketching will sort this one. The LHS values at x= (0,-6,-12,-18) are all 0. The polynomial is a quartic (with a +ve x^4 coeff) so will have 'w' form and there will 2 real roots at x>0 and x< -18 and maybe 2 more. The symmetry of the four zero crossing coordinates means that roots will have the form of a ± \pm b and a ± \pm c where a = -9 (-9 being the mid-way between 0 and -18). x = -9 is the maximum between the two minima and its value is 729 which is less than 1729 this means that there are only two real roots. Since the roots are -9+b and -9-b so the sum is 2 x -9 = -18.

A bit of sketching and looking can save a great deal of maths.

Ankit Chabarwal
Jul 28, 2016

Put X=x+9 Then the given equation reduces to (X-9)(X-3)(X+3)(X+9)=1729 => (X^2 - 9^2)(X^2 - 3^2)=1729 Put X^2=t => (t - 81)(t - 9)=1729 => t^2 - 90t - 1000=0 By quadratic formula t=100,-10=(x+9)^2 =>x=1,-19

So, the sum of all(2) real roots is -18.

0 pending reports

×

Problem Loading...

Note Loading...

Set Loading...